Warum scheinen diese Berechnungen von EM-Feldern für einen Magneten und eine Drahtschleife inkonsistent zu sein?

Angenommen, Sie haben eine leitende kreisförmige Drahtschleife und einen Magneten, die sich aufeinander zu bewegen. Sie bewegen sich entlang der z Richtung mit nichtrelativistisch konstanter Geschwindigkeit v . Lassen Sie die B Feld des Magneten in seinem Bezugssystem parallel zu sein z Richtung:

B = ( 0 , 0 , z ) ,
also die Stärke von B nimmt linear ab z . Die Flächennormale der von der Drahtschleife umschlossenen Fläche zeige in die z Richtung auch. Hier nun mein Problem: Wenn ich die auf Ladungen im Draht wirkenden Felder und Kräfte berechnen möchte, erhalte ich unterschiedliche Ergebnisse.

Im Bezugssystem des Magneten herrscht ein statisches Magnetfeld B = ( 0 , 0 , z ) , und die Drahtschleife bewegt sich mit konstanter Geschwindigkeit v = ( 0 , 0 , 1 ) zum Magneten. In diesem Fall gibt es keine E Feld, also die resultierende Kraft auf Ladungen ist F = Q v × B . Seit v Und B parallel sind, ist das Kreuzprodukt 0, also gibt es keine Kraft.

Wenn ich auf das Referenzsystem des Drahtes schaue , gibt es ein zeitlich veränderliches Magnetfeld, das ein elektrisches Feld erzeugt. Hier verstehe ich es nicht ganz. Das elektrische Feld wird

E ' = v × B
laut Wikipedia (Gamma ist ungefähr 1). In diesem Fall
E ' = 0 v = 0 F = Q ( E + v × B ) = 0
genau wie im anderen Bezugssystem.

Aber wenn ich versuche, stattdessen die Faraday-Gleichung zu verwenden , sieht die Drahtschleife ein zeitlich veränderliches Magnetfeld mit einem Oberflächenintegral ungleich Null, also muss es ein nicht-konservatives, ungleich Null geben E Feld, das auf die Ladungen einwirkt.

Ich habe das mit verschiedenen Magnetfeldern versucht und jedes Mal scheitern meine Berechnungen kläglich. Irgendwo fehlt etwas, aber ich weiß nicht warum.

Übrigens, gute Präsentation für ein Hausaufgabenproblem.
Danke, @Daniel, ich habe die Tags bearbeitet, sodass sie jetzt "Hausaufgaben und Übungen" enthalten :) Ich bin neu hier, also wusste ich nicht, dass dieses Tag existiert.
Je weiter man sich vom Magneten entfernt, desto geringer wird der magnetische Fluss und umgekehrt. In beiden Fällen wirkt also ein induziertes elektrisches Feld auf die Schleife, das einen Strom verursacht, der dann bewirkt, dass die Schleife eine Netto-Lorentz-Kraft erfährt. Das ist wichtig, die Flussänderung an der Schleife aufgrund der relativen Bewegung des Magneten und der Schleife, die in beiden Referenzrahmen äquivalent ist.
IE, Sie können nur das Faradaysche Gesetz verwenden, da es sich um eine verknüpfte Flussänderung handelt, die Sie ignoriert und nur reine Lorentzkräfte berücksichtigt haben.
Zu Ihrer Information, die magnetische Flussintensität fällt ebenfalls ab 1 / R 3 entlang der Achse des magnetischen Dipols.
Wie Sie sehen können, habe ich auch das Faradaysche Gesetz berücksichtigt, da entsteht das Problem. Sie kann aber nur genutzt werden, wenn eine zeitlich veränderliche Flussdichte vorliegt. Im Magnetreferenzrahmen können Sie deutlich sehen, dass es keinen gibt. Stattdessen wird in diesem Referenzrahmen normalerweise eine bewegungsinduzierte EMK berechnet, die in diesem Fall o ist, weil die Bewegung parallel zu den Flusslinien ist.
Vielen Dank auch für die Informationen über die magnetische Flussdichte, aber diese Gesetze sollten für alle Arten von Vektorfeldern funktionieren, ich habe für dieses Beispiel nur ein wirklich einfaches ausgewählt. Das Hauptproblem ist immer noch das gleiche: Das Wechseln von Referenzrahmen führt zur Verwendung unterschiedlicher Gleichungen: Magnetreferenzrahmen - bewegungsinduzierte EMF, Drahtreferenzrahmen - Faradaysches Gesetz, und sie geben mir unterschiedliche Lösungen.
Übrigens, Bewegungs-EMK E = Bv xl, Lorentz-Kraft F = iB xl, wobei Strom I = E / R, wobei R der Widerstand der Schleife und l der Durchmesser des Drahtes ist (berücksichtigen Sie l doppelt so wie die Schleife durch seinen Durchmesser in 2 Teile geteilt)

Antworten (1)

Sie übersehen die Tatsache, dass Ihre Situation, wie Sie sie definiert haben, physikalisch unmöglich ist. Denken Sie daran, dass eine von Maxwells Gleichungen ist B = 0 , aber Ihre Magnetfeldkonfiguration B ( 0 , 0 , z ) befriedigt dies nicht. Angesichts dessen ist es nicht verwunderlich, dass die anderen Gleichungen von Maxwell inkonsistente Ergebnisse liefern.

Übrigens, ( 0 , 0 , z ) ist nicht das Magnetfeld eines Punktmagneten. Sie würden eine ausgedehnte, kontinuierliche Verteilung der magnetischen Ladung benötigen, um dieses Feld zu erhalten. Wenn Sie davon ausgehen, dass magnetische Monopole existieren und Sie sie so verteilen können, dass diese Feldkonfiguration erzeugt wird, hätten Sie im Referenzrahmen des Drahtes eine "Flüssigkeit" magnetischer Monopole in Bewegung, die das elektrische Feld erklären würde.

Vielen Dank!! Ich habe versucht, dieses Problem so sehr zu vereinfachen, dass ich vergessen habe zu prüfen, ob dies überhaupt möglich ist oder nicht. Ich werde versuchen, dieses Problem in einem realen Magnetfeld zu formulieren und die Berechnungen durchzuführen. Ich hoffe, dass ich keine widersprüchlichen Lösungen bekomme.
Das wirst du nicht, wenn du es richtig machst. Ich habe viele Leute gesehen, die diesen Fehler gemacht haben (mich eingeschlossen), und es funktioniert immer sauber, wenn Sie sicherstellen, dass alle richtigen Einschränkungen erfüllt sind.
Es ist wirklich gut zu hören, dass ich nicht die einzige bin, die solche Fehler macht. Ich werde in Zukunft vorsichtiger sein und alle Maxwell-Gleichungen berücksichtigen, wenn ich versuche, ein Problem zu erstellen.